majoration du terme général d'une suite

Bonjour,

Je sèche lamentablement sur la question suivante :

On donne :$u_0>0$, $u_1>0$ et $u_{n+1}=u_n+\frac{1}{n(n+1)}u_{n-1}$
Montrer qu'il existe M positif tel que $u_n \leq M(1-1/n)$.

La suite est bien composée de nombres strictement positifs, elle est strictement croissante, OK.
En fait je parviens bien à prouver ce résultat en montrant que cette suite converge (en utilisant le fait que le produit pour k allant de 1 à n des $(1+1/k^2)$ converge à l'aide d'un passage au ln et d'une étude de série à termes positifs) mais la position de cet exercice dans l'ouvrage où je l'ai trouvé suggère une solution plus élémentaire.

Si quelqu'un peut m'aider !

Réponses

  • Ne pas oublier la décomposition en élément simple de $\frac{1}{n(n+1)}=\frac{1}{n}-\frac{1}{(n+1)}$
  • Si je ne m'abuse cela fonctionne par récurrence (à partir du rang 2).
  • Une récurrence semble bien faire l'affaire. On peut essayer de montrer l'inégalité sous la forme $nu_n \leq M(n-1)$.
  • En effet, récurrence.
    D'où vient cet exercice ?
  • Cet exercice provient du manuel d'analyse (volume 1) de G.Debeaumarché (ex 14 p 182).
    Merci pour vos réponses.

    Effectivement, une récurrence aven un peu de calcul s'avère efficace, je cherchais quelque chose de plus subtil.

    En revanche, la décomposition en élément simple ne m'a pas été utile et c'est justement le $\frac{1}{n(n+1)}$ qui m'orientait vers une solution plus simple (avec télescopage ou quelque chose du genre), mais je n'ai pas trouvé de solution qui exploite ceci.
  • Très mauvais exercice, à plus d'un égard.

    La solution que tu as proposée dans ton premier message me semble la plus naturelle (évidemment avec la culture qui est la tienne, la mienne).

    L'inégalité n'a aucun intérêt si M n'est pas la limite de la suite.

    Et s'il s'agit juste de faire des gammes sur la récurrence, pourquoi faire le mystérieux et ne pas donner la valeur de M ?
  • En effet c'est presque l'initialisation de la récurrence qui pose le plus de problèmes... et la décomposition en éléments simples accélère grandement la preuve de l'hérédité. La présence du M sert peut-être à donner l'impression que cet exercice est plus difficile qu'il ne l'est vraiment ?
  • Voici comment je rédige ce raisonnement par récurrence.
    L'assertion $P(n)$ à démontrer pour $n\geq 2$ est : « $u_{n}\leq \frac{n-1}{n}M$ et $u_{n+1}\leq \frac{n}{n+1}M$ ».
    Si $P(n)$ est vraie, alors :
    $u_{n+2}=u_{n+1}+\frac{1}{(n+1)(n+2)}u_{n}\leq \frac{n}{n+1}M+\frac{1}{(n+1)(n+2)}\cdot \frac{n-1}{n}M=\frac{n^{3}+2n^{2}+n-1}{n(n+1)(n+2)}M$.
    Il faut prouver : $\frac{n^{3}+2n^{2}+n-1}{n(n+1)(n+2)}\leq \frac{n+1}{n+2}$, qui équivaut à : $n^{3}+2n^{2}+n-1\leq n(n+1)^{2}$, vrai.
    L'assertion $P(2)$ est : « $u_{2}\leq \frac{1}{2}M$ et $u_{3}\leq \frac{2}{3}M$ » . Elle est vraie en prenant : $M=\max (2u_{2},\frac{3}{2}u_{3})$.
    Bonne journée.
    Fr. Ch.
    14/09/2017
  • Une autre manière de faire et je ne prétends pas que c'est la solution la plus simple.

    J'ai modifié mon message initial il utilisait implicitement que $u_1>u_0$ ce qui ne permettait pas d'affirmer que (1) est vraie pour tout $n\geq 1$ si on se place dans l'hypothèse que $u_1,u_0$ sont quelconques mais strictement positifs.
    A priori, la suite n'est pas croissante et on peut avoir, par exemple, $u_2<u_0$.

    On a pour tout $n\geq 1$,
    $u_{n+1}=u_n+\frac{1}{n(n+1)}u_{n-1}$

    ( En particulier, pour tout $n\geq 1$, $u_{n+1}\geq u_n$. )

    Puisque $(u_n)_{n\geq 1}$ est croissante on a pour tout $n\geq 2$:
    $\displaystyle u_{n+1}\leq \left(1+\frac{1}{n(n+1)}\right)u_n$ (1).

    Puisque $(u_n)$ est strictement positive on a pour tout $n\geq 2$:
    $\displaystyle \frac{u_{n+1}}{u_n}\leq 1+\frac{1}{n(n+1)}$

    Puisque $(u_n)$ est strictement positive et la fonction logarithme est croissante sur son domaine de définition,
    on a pour tout $n\geq 2$:
    $\displaystyle \ln\left(\frac{u_{n+1}}{u_n}\right)\leq \ln\left(1+\frac{1}{n(n+1)}\right)$

    Puisque pour tout $x>0$ , $\ln(1+x)\leq x$,
    $\displaystyle \ln\left(\frac{u_{n+1}}{u_n}\right)\leq \frac{1}{n(n+1)}$

    Soit $m\geq 3$.

    On a:
    $\displaystyle \sum_{n=2}^{m-1} \ln\left(\frac{u_{n+1}}{u_n}\right)\leq \sum_{n=2}^{m-1} \frac{1}{n(n+1)}$

    Or,
    $\displaystyle \sum_{n=2}^{m-1} \ln\left(\frac{u_{n+1}}{u_n}\right)=\ln\left(\frac{u_{m}}{u_2}\right)$

    et,
    $\displaystyle \sum_{n=2}^{m-1} \frac{1}{n(n+1)}=\dfrac{1}{2}-\dfrac{1}{m+1}$

    (la somme est une somme télescopique puisque $\frac{1}{n(n+1)}=\frac{1}{n}-\frac{1}{n+1}$ )

    Donc, pour tout $m\geq 3$,
    $\displaystyle \ln\left(\frac{u_{m}}{u_2}\right)\leq \frac{1}{2}-\frac{1}{m+1}$

    La fonction exponentielle est croissante sur son domaine de définition,
    Donc, pour tout $m\geq 3$,
    $\displaystyle \frac{u_{m}}{u_2}\leq \exp\left(\frac{1}{2}-\frac{1}{m+1}\right)$
    et $u_2>0$ ainsi, pour tout $m\geq 3$,
    $\displaystyle u_{m}\leq u_2\exp\left(\frac{1}{2}-\frac{1}{m+1}\right)$

    On peut montrer que,
    la suite définie pour tout $n>1$ par ,
    $\displaystyle \frac{\exp\left(\frac{1}{2}-\frac{1}{n+1}\right)}{\sqrt{\text{e}}\left(1-\frac{1}{n}\right)}$ converge vers $1$.

    Il existe donc $A>0$ tel que pour tout $n>1$ on a:
    $\displaystyle \exp\left(\frac{1}{2}-\frac{1}{n+1}\right)\leq A\sqrt{\text{e}}\left(1-\frac{1}{n}\right)$

    On peut donc en déduire que pour tout $n\geq 3$,
    $\displaystyle u_n\leq u_2\times A\sqrt{\text{e}}\left(1-\frac{1}{n}\right)$

    Or,
    $u_2=u_1+\dfrac{1}{2}u_0=\left(2u_1+u_0\right)\left(1-\frac{1}{2}\right)$

    Si on prend $M=\text{Max}\left(2u_1+u_0,u_2\times A\sqrt{\text{e}}\right)$ on a pour tout $n\geq 2$,
    $\displaystyle u_n\leq M\left(1-\frac{1}{n}\right)$

    J'espère qu'il n'y a plus d'erreurs.
  • chaurien a écrit:
    Il faut prouver :

    Il [large]suffit[/large] de prouver ;-)
    Aide les autres comme toi-même car ils sont toi, ils sont vraiment toi
  • Peut on savoir cc ta preuve ( toi qui n'aime pas les calculs)
    Le 😄 Farceur


  • @ cc
    En effet, j'ai eu une hésitation. Sur le plan purement logique tu as raison, et vu ta compétence dans ce domaine ce n'est pas étonnant. Mais je me suis dit : « je dois prouver ceci », autrement dit : « il me faut prouver ceci ». Ce n'était pas un discours logique, mais heuristique. Alors, j'ai raison même quand j'ai tort ;-) ?
    Bravo pour ta remarque et bonne journée.
    Fr. Ch.
  • Moi je le trouve intéressant cet exercice.

    La suite $u_n$ est évidemment positive, et elle est donc croissante, et il nous « faut » prouver qu'elle est majorée : c'est là le point important.

    Ici nous touchons à la stratégie de la récurrence. Dans « $P(n)\Rightarrow P(n+1)$ » , l'assertion $P(n)$ est à la fois l'hypothèse et la conclusion, en quelque sorte. Alors si nous ne mettons pas assez de choses dans $P(n)$ nous n'aurons pas assez d'hypothèse, mais si nous en mettons trop, nous aurons trop de conclusion. Dans le cas présent, si nous avions pris pour assertion $ P(n)$ à prouver : « $u_{n}\leq M$ et $u_{n+1}\leq M$ », nous n'aurions pas assez d'hypothèse et n'aurions pas pu mettre en œuvre notre récurrence. Heureusement l'énoncé, au lieu de nous demander « prouver que la suite $u_n$ est majorée », nous sert une propriété plus précise qui permet la récurrence, comme j'ai montré dans un précédent message. Et la majoration trouvée prouve que la suite $u_n$ est majorée.

    Ainsi, notre suite $u_n$ a une limite $L>0$, et la forme de la relation de récurrence permet d'affirmer : $u_{n}=L-\frac{L}{n}+o(\frac{1}{n})$ quand $n\rightarrow +\infty $, et l'on peut pousser le développement asymptotique si l'on veut.

    En fait, cette dernière conclusion s'étend à toute suite régie par la même relation de récurrence, mais avec deux valeurs initiales $u_0$ et $u_1$ quelconques, réelles ou complexes. En effet, cette relation de récurrence est linéaire d'ordre $2$, et qu'elle soit à coefficients non constants n'empêche pas l'ensemble de ces suites d'être un espace vectoriel de dimension $2$, réel ou complexe selon les hypothèses. Et un tel espace a une base constituée de deux suites dont les deux premiers termes sont pour chacune des réels strictement positifs. D'ailleurs, on peut légèrement affaiblir l'hypothèse initiale « $u_{0}>0$ et $u_{1} >0$ » en : « $u_{0}\geq 0$ et $u_{1}\geq 0$ et $(u_{0},u_{1})\neq (0,0)$ », la démonstration de récurrence précédente reste valable, et l'on peut prendre pour base la base canonique.

    Bonne journée.
    Fr. Ch.
  • Bonjour,
    Chaurien ecrivait : il nous « faut » prouver qu'elle est majorée : c'est là le point important...Heureusement l'énoncé, au lieu de nous demander « prouver que la suite un est majorée »....

    Par le télescopage de FDP la majoration de la suite est immédiate
    Le 😄 Farceur


  • Coucou de mon téléphone : oui bien sûr je ne faisais que partager une émotion instinctive je sais que chaurien sait. @gebrane: pardon de te décevoir mais je n'ai "bien évidemment aucune preuve" à proposer (je ne prouve sue ce dont je suis convaincu et l'énoncé m'inspire "je ne sais pas" quand je le lis pour la première fois). Bien entendu je serais probablement convaincu par les preuves des autres intervenants si je me mettais en chantier de lire les calculs... Mais j'en aurais pour jusqu'à demain matin :-D Ce n'est pas tout a fait mon projet de soirée.
    Aide les autres comme toi-même car ils sont toi, ils sont vraiment toi
Connectez-vous ou Inscrivez-vous pour répondre.